Vous êtes sur la page 1sur 11

Question #1: Data Sufficiency

If w < x < y < z, is the average (arithmetic mean) of w, x, y, and z greater than y?
(1) w + z = x + y
(2) w = 2 and z = 18
A. Statement (1) ALONE is sufficient, but statement (2) alone is not sufficient.
B. Statement (2) ALONE is sufficient, but statement (1) alone is not sufficient.
C. BOTH statements TOGETHER are sufficient, but NEITHER statement ALONE is sufficient.
D. EACH statement ALONE is sufficient.
E. Statements (1) and (2) TOGETHER are NOT sufficient.
Answer:
(A) is correct.
Data Sufficiency will be unfamiliar if you have never engaged with the GMAT. But don’t worry ﹘
with the right strategies and some practice, you will master this question type.
The goal in Data Sufficiency is not actually to solve the problem, but to determine whether or not
you have sufficient information to do so. (The good news is that there may be less calculation
involved in answering a Data Sufficiency question than in answering a Problem Solving question.) A
key step in getting to the correct answer is thinking through what sort of information would be
sufficient to arrive at a solution before you investigate the two statements.
First, determine that this is a yes/no question (rather than a value question). Then simplify the
question so that you know what you need to find to determine sufficiency.
If you knew what the 4 variables were, that would certainly be sufficient. But also look at the actual
question that’s being asked:
Is (w + x + y + z)/4 > y?
Stated differently:
Is w + x + y + z > 4y?
In other words:
Is w + x + z > 3y?
This is the actual question being asked.
Statement 1 tells us that w + z = x + y.
If we plug that into the new simplified question, we get:
Is w + x + z > 3y ? Plug in x + y for w + z:
Is 2x + y > 3y ?
Is 2x > 2y ?
Is x > y ? We were given that w < x < y < z so the answer to this question is ALWAYS NO.
So Statement 1 is sufficient - Eliminate B, C and E. On to Statement 2.
Statement 2 tells us the values of two variables. At first glance, it looks like concrete information, but
plug it in and simplify:
Is w + x + z > 3y ? Plug in for w and z.
Is 2 + x + 18 > 3y?
Is x + 20 > 3y ?
Well, if x = 4 and y = 5 then this is true because 24 > 15. That's a "yes."
But what if x = 4 and y = 15? 24 is not greater than 45, so that's a "no."
Sometimes yes and sometimes no: definitely insufficient. The correct answer choice is A: Statement
1 only.

Question #2: Problem Solving


If n = 66 and n^4 = m x 6 x 22 x 33 x 198, what is the value of m?
A. 11
B. 22
C. 33
D. 44
E. 66
Answer:
Whenever you're faced with very large numbers in the form of exponents or several numbers
multiplied together, consider using prime factorization. Here's how to use prime factorization to
make short work of this question:
n = 66 = 2 x 3 x 11
n x n x n x n = (2 x 3 x 11) x (2 x 3 x 11) x (2 x 3 x 11) x (2 x 3 x 11)
m x 6 x 22 x 33 x 198 = m x (2 x 3) x (2 x 11) x (3 x 11) x (2 x 3 x 3 x 11)
Now set n x n x n x n equal to m x 6 x 22 x 33 x 198.
(2 x 3 x 11) x (2 x 3 x 11) x (2 x 3 x 11) x (2 x 3 x 11) = m x (2 x 3) x (2 x 11) x (3 x 11) x (2 x 3 x 3 x
11)
Now rearrange the factors:
2 x 2 x 2 x 2 x 3 x 3 x 3 x 3 x 11 x 11 x 11 x11
Equals
2 x 2 x 2 x 3 x 3 x 3 x 3 x 11 x 11 x 11 x m
Anything they have in common cancels out:
2 x 2 x 2 x 2 x 3 x 3 x 3 x 3 x 11 x 11 x 11 x 11
Equals: 2 x 2 x 2 x 3 x 3 x 3 x 3 x 11 x 11 x 11 x m
All the factors cancel out leaving only m on one line and 2 x 11 on the other. Since both lines are
equal, m = 2 x 11 = 22. BOOM!
Choice B is correct.

Question #2: Problem Solving


If n = 66 and n^4 = m x 6 x 22 x 33 x 198, what is the value of m?
A. 11
B. 22
C. 33
D. 44
E. 66
Answer:
Whenever you're faced with very large numbers in the form of exponents or several numbers
multiplied together, consider using prime factorization. Here's how to use prime factorization to
make short work of this question:
n = 66 = 2 x 3 x 11
n x n x n x n = (2 x 3 x 11) x (2 x 3 x 11) x (2 x 3 x 11) x (2 x 3 x 11)
m x 6 x 22 x 33 x 198 = m x (2 x 3) x (2 x 11) x (3 x 11) x (2 x 3 x 3 x 11)
Now set n x n x n x n equal to m x 6 x 22 x 33 x 198.
(2 x 3 x 11) x (2 x 3 x 11) x (2 x 3 x 11) x (2 x 3 x 11) = m x (2 x 3) x (2 x 11) x (3 x 11) x (2 x 3 x 3 x
11)
Now rearrange the factors:
2 x 2 x 2 x 2 x 3 x 3 x 3 x 3 x 11 x 11 x 11 x11
Equals
2 x 2 x 2 x 3 x 3 x 3 x 3 x 11 x 11 x 11 x m
Anything they have in common cancels out:
2 x 2 x 2 x 2 x 3 x 3 x 3 x 3 x 11 x 11 x 11 x 11
Equals: 2 x 2 x 2 x 3 x 3 x 3 x 3 x 11 x 11 x 11 x m
All the factors cancel out leaving only m on one line and 2 x 11 on the other. Since both lines are
equal, m = 2 x 11 = 22. BOOM!
Choice B is correct.

Question #4: Sentence Correction


In his search for an effective method of motivating his students to practice, the piano teacher
found that the more he reminded his students of their possible embarrassment in the upcoming
recital, their willingness to prepare themselves in advance increased.
A. their willingness to prepare themselves in advance increased
B. their willingness to be prepared in advance was increasing
C. their willingness for advanced preparation was greater
D. the greater their willingness became to prepare themselves in advance
E. the greater their willingness for advanced preparation increased
Answer:
(D) is correct.
Sentence Corrections questions can be challenging for many test takers. The good news is that
the GMAT tests several recurring grammar issues repeatedly. Learn to spot those and you’ll be in
great shape.
The issue featured in this sentence is parallelism. As written, this sentence presents an unparallel
comparison. When a comparison begins with "the more…," it must be followed by a parallel
comparative. Think of the phrase "the more, the merrier." Choices (A), (B) and (C) can be
eliminated because they do not begin with comparatives.
Choices (D) and (E) both begin with the comparative, "the greater," but (E) introduces the
redundancy "the greater their willingness…increased." Moreover, (E)'s "advanced preparation"
does not convey the idea of preparing in advance that the sentence requires.
Choice (D) corrects the unparallel comparison without introducing any new problems and is,
therefore, the correct answer

question #1

If n is an integer, is n - 1 even?
(1) n - 2 is odd
(2) n + 1 is even
(A) Statement (1) ALONE is sufficient, but statement (2) alone is not sufficient.
(B) Statement (2) ALONE is sufficient, but statement (1) alone is not sufficient.
(C) BOTH statements TOGETHER are sufficient, but NEITHER statement
ALONE is sufficient.
(D) EACH statement ALONE is sufficient.
(E) Statements (1) and (2) TOGETHER are NOT sufficient to answer the
question asked, and additional data are needed.
Answer:

(D) is correct.
This question asks whether the information in one or both statements is
sufficient to answer a yes or no question. The interesting thing about this type
of Data Sufficiency question is that a "no" is just as conclusive as a "yes;" as
long as the answer is always a "yes" or always a "no," the information is
sufficient. The information is insufficient when using that information leaves
both "yes" and "no" answers possible (in other words, when the answer is
"maybe," the information is insufficient).
If n - 2 is odd, then we know that n itself must be odd. Not sure about that?
Make up some numbers and test it out. So if n is odd, then n - 1 must be even.
Statement (1) is sufficient.
Statement (2) tells us that n + 1 is even. That means that n is odd, which
means that n - 1 is also even. Statement (2) is also sufficient!
The credited response is D: Each statement, when considered
independently, leads to an absolute answer in all circumstances (for the
record here, both "yes").
Question #2

If c = 4x + 5 and r = 21 - 2x, then for what value of x is c = r?


A. 3⁄8
B. 8⁄3
C. 13⁄3
D. 8
E. 13
Answer:

(B) is correct.
⁄3 is the credited response.
8

The question states that c = r. So 4x + 5 must equal 21 - 2x. Set the


expressions equal to one another and solve for the value of x:
4x + 5 = 21 - 2x
6x = 16
x = 16⁄6 = 8⁄3
Question #3

Apartments are labeled 2 through 56 consecutively on the first floor of a


building. What is the probability that a tenant living there will have an
apartment with a tens digit of 3?
A. 9⁄55
B. 2⁄11
C. 5⁄27
D. 10
E. 55
Answer:

(B) is correct.
This is a more basic probability question. First, remember that to calculate the
number of apartments from 2 through 56, we should combine:
56 - 2 + 1 = 55 apartments. We add 1 because we're including both endpoints
in the calculation; normal subtraction only counts the distance moved from one
endpoint to another on a number line, so it essentially leaves one of two
endpoints out.
Next we consider the element of probability. The formula for probability is
number of desired outcomes⁄total possibilities, and keep in mind that since probability expresses
likelihood, it is always somewhere between 0 (no chance) and 1 (100%
certainty). It's most commonly presented as a fraction.
In this scenario, there are 10 apartments that begin with the tens digit of "3"
(30 - 39), so we have a total of 10 desirable outcomes. And we already
determined that there are 55 total possibilities (55 possible apartments labeled
2 through 56), so 10⁄55 = 2⁄11

Question #4

4. Sally writes her brother a check for $55, which represents her repayment of
20% of an earlier loan. If this is Sally's first payment on the loan, and if her
brother charges her no interest, how much does she still owe him?
A. $11
B. $44
C. $220
D. $275
E. $1100
Answer:

(C) is correct.
$220 is the credited response.
You could solve algebraically, by first finding the amount loaned, and then the
balance, but it is much faster and easier to solve for the balance directly:
If the $55 payment is equal to 20% of the loan, then the balance is equal to
80%. 80% is four times 20%, so the balance is 4*$55, or $220.
You could also have done this with fractions, using 1/5 and 4/5 in place of
20% and 80%.
Question #5

4.7☆◎3
If ☆ and ◎ each represent single digits in the decimal above, what digit does
☆ represent?
(1) When the decimal is rounded to the nearest tenth, 4.8 is the result.
(2) When the decimal is rounded to the nearest hundredth, 4.77 is the result.
(A) Statement (1) ALONE is sufficient, but statement (2) alone is not sufficient.
(B) Statement (2) ALONE is sufficient, but statement (1) alone is not sufficient.
(C) BOTH statements TOGETHER are sufficient, but NEITHER statement
ALONE is sufficient.
(D) EACH statement ALONE is sufficient.
(E) Statements (1) and (2) TOGETHER are NOT sufficient to answer the
question asked, and additional data are needed.
Answer:

(E) is correct.
This question asks whether the information in one or both statements is
sufficient to find a specific value.
By statement (1), we know that the hundredths digit (represented by ☆) must
be equal to 5, 6, 7, 8, or 9. We cannot narrow it down further than that, so
statement (1) is not sufficient.
Statement (2) shows us the hundredths digit, but it has been rounded. Since
we do not know the thousandths digit, we cannot be sure whether the
hundredths digit is indeed 7, or if it was rounded up from 6. Because we
cannot be sure of ☆'s value, this is not sufficient.
Even combined, we do not have enough information to answer the question.
The credited response is E: The statements don't provide sufficient
information to find the value in question, not even when considered
together.

Question #6

For members of various bygone Inuit cultures in the far northern hemisphere,
sled dogs were a vital element of daily life, to provide feasible transportation
across the vast arctic landscape.
A. to provide
B. a means to provide
C. as a means providing
D. providing
E. as a provision of
Answer:

(D) is correct.
Although our focus is tightly drawn in this sentence, a complex sentence
structure makes this question tougher than it might first appear. The sentence
could logically end at the comma, but we’ve got a phrase tacked on at the
end, and the underlined portion begins that phrase. Since there’s a comma,
that portion of the sentence must describe the subject of the phrase that
precedes it, “sled dogs.” And although there’s a verb in the underlined portion,
it’s not being used as a verb... at least not outright. In situations like this, use
logic to guide your decision. Start with the longer choices to see if you can rule
them out. Calling sled dogs “a means” is awkward and just doesn’t make
sense, and it’s not proper idiom. Finally, we have to choose the proper verb
form from among the remaining choices, so think about time.

Question #7

Not all stars emit energy as visible light. Some stars emit energy at high
frequencies outside the realm of visible light, such as gamma rays. Other stars,
particularly in the Milky Way galaxy, will emit energy in the form of invisible X-
rays.
If the statements to the left are true, which of the following conclusions is best
supported?
A. A thorough analysis of visible light emissions is insufficient to monitor star
energy emissions.
B. Gamma rays are also a common type of energy emission in the Milky Way
galaxy.
C. X-rays emissions found outside the Milky Way galaxy are always paired with
visible light emissions from the same star.
D. Dying stars are more likely to emit gamma rays.
E. Telescopes capable of observing gamma rays are incapable of observing
visible light frequencies.
Answer:

(A) is correct.
The passage presents statements about both gamma rays and Xrays. The best
conclusion will offer a general statement that applies to both, rather than a
specific statement about either one or the other. Both gamma rays and Xrays
are outside the spectrum of visible light, so it is true that measuring visible light
emissions alone is insufficient.
Question #8

Both Manny Industries and Scotch Industries manufacture the same type of
metal gaskets. Raw material supplies represent 12 percent of the cost to
manufacture gaskets at each company. Manny Industries should lower its raw
material supplies costs to gain a competitive advantage over Scotch
Industries.
Which of the following, if true, casts the most doubt on the argument stated
above?
A. The cost of raw materials has steadily climbed over the past decade, mostly
due to increased freight costs.
B. Unionization makes it difficult for metal gasket manufacturers to lower
wages or fire existing employees.
C. In order to lower its raw materials costs, Manny Industries would need to
sacrifice the quality of their finished product.
D. Scotch Industries has outperformed Manny Industries by a small margin for
the last three quarters.
E. The main raw materials supplier for Manny Industries also caters to Scotch
Industries.
Answer:

(C) is correct.
It is easy to say Manny Industries should cut costs to gain an advantage, but
what are they losing by cutting costs? If they sacrifice quality, they may
actually lose their competitive edge against competitor with a product of
superior quality.

Question #9

Very often one hears a person who has become famous or successful in their
field or endeavor say that they "knew it all along." Although there may be a bit
of rosy hindsight in such a claim, the fact remains that self-confidence is an
indispensable component of success.
The author of the following passage would most likely agree with which of the
statements below?
A. It is not possible to become successful in one's field of endeavor unless one
has a certain amount of self-confidence.
B. Self-confidence is an innate trait, and cannot be influenced by such factors
as environment and circumstance.
C. What famous and successful people claim, and what is actually the case,
are very often two different things.
D. Although there may be successful people who are self-confident, there are
many more self-confident people who are not successful.
E. Once a person is famous, self-confidence becomes less important than
quotidian considerations such as careful planning and foresight.
Answer:

(A) is correct.
The author of the argument says that although there is a bit of after-the-fact
confidence in statements like “I knew it all along,” eventual
success requires self-confidence. The author, then, would agree with (A),
which says exactly this: Success requires self-confidence. We have no idea
what the author would think of (B), (C) plays off of the author’s small
concession to rosy hindsight but grossly exaggerates the conclusion, (D)
suggests the author’s views on whether or not most self-confident people are
successful. For all we know, the author may believe that very few self-
confident people are unsuccessful. And finally, (E) suggests that the author
considers self-confidence more important in achieving success than
maintaining it. Again, this is another issue about which we don’t know the
author’s thoughts.

Question #10

The Christmas holiday brought a brief respite to opposing armies during World
War I in that an unofficial ceasefire spread along the Western Front on
Christmas Eve, 1914.
A. armies during World War I in that an unofficial ceasefire spread along the
Western Front on Christmas Eve, 1914
B. armies during World War I, spreading an unofficial ceasefire along the
Western Front on Christmas Eve, 1914
C. armies during World War I when they spread an unofficial ceasefire along
the Western Front on Christmas Eve, 1914
D. armies during World War I, for an unofficial ceasefire spread along the
Western Front on Christmas Eve, 1914
E. armies during World War I by the spread of an unofficial ceasefire along the
Western Front on Christmas Eve, 1914
Answer:

(D) is correct.
Both proper idiom and the logic of the sentence must guide our selection of
the correct answer choice. We need to consider how the second portion of the
sentence relates to the first portion: the second portion describes how the
Christmas holiday brought respite to the opposing armies. In a cause-and-
effect relationship, verbal clues help to clarify the relationship. We're looking
for a conjunction that suggests a cause-effect relationship.

Vous aimerez peut-être aussi